LSAT and Law School Admissions Forum

Get expert LSAT preparation and law school admissions advice from PowerScore Test Preparation.

 Administrator
PowerScore Staff
  • PowerScore Staff
  • Posts: 8927
  • Joined: Feb 02, 2011
|
#26041
Complete Question Explanation
(The complete setup for this game can be found here: lsat/viewtopic.php?f=107&t=7219)

The correct answer choice is (A)

If Z2 includes an H subzone, then according to the fourth rule of the game, Z2 does not include an I subzone:
oct12_game_4_#23_diagram_1.png
In addition, this question stem establishes that there are a total of 3 R subzones. In compliance with the first rule, none of these subzones can be added to Z1. Additionally, at most one R can be added to Z2 (third rule). Therefore, either all three R subzones will be added to Z3, or else two of them will be added to Z3, and one—to Z2:
oct12_game_4_#23_diagram_2.png
Finally, recall that no more than one R subzone is allowed in any zone containing an H subzone (third rule). From this we can infer that Z3 cannot contain an H subzone:
oct12_game_4_#23_diagram_3.png
The setups above are sufficient to eliminate every answer choice except for answer choice (A), which could be true and is therefore correct.
 reop6780
  • Posts: 265
  • Joined: Jul 27, 2013
|
#16850
The wording of the question sterm was a bit confusing while I worked on it.

I pretented "in all" was absent, and thought RRR spreads somewhere in different zones. (2 and 3)

However, while the entire rules and questions were confusing in regard with their language, I was not sure whether Q23 was meant RRR in one zone or what.

Does it make any difference if the question stem omits "in all" ?
 Robert Carroll
PowerScore Staff
  • PowerScore Staff
  • Posts: 1774
  • Joined: Dec 06, 2013
|
#16904
reop,

"In all" just means that there are exactly three subzones for retail use somewhere, not necessarily all in one zone. As Z1 cannot have these, they must be spread among Z2 and Z3; but since Z2 has a subzone for housing, it can't have more than 1 of the 3 retail.

Robert Carroll
User avatar
 rjulien91
  • Posts: 10
  • Joined: Jan 24, 2023
|
#99206
You are still expected to place the remaining 2 Hs and 3Is right?
Administrator wrote: Sun Jan 20, 2013 12:00 am Complete Question Explanation
(The complete setup for this game can be found here: lsat/viewtopic.php?f=107&t=7219)

The correct answer choice is (A)

If Z2 includes an H subzone, then according to the fourth rule of the game, Z2 does not include an I subzone:

oct12_game_4_#23_diagram_1.png

In addition, this question stem establishes that there are a total of 3 R subzones. In compliance with the first rule, none of these subzones can be added to Z1. Additionally, at most one R can be added to Z2 (third rule). Therefore, either all three R subzones will be added to Z3, or else two of them will be added to Z3, and one—to Z2:

oct12_game_4_#23_diagram_2.png

Finally, recall that no more than one R subzone is allowed in any zone containing an H subzone (third rule). From this we can infer that Z3 cannot contain an H subzone:

oct12_game_4_#23_diagram_3.png

The setups above are sufficient to eliminate every answer choice except for answer choice (A), which could be true and is therefore correct.
 Adam Tyson
PowerScore Staff
  • PowerScore Staff
  • Posts: 5153
  • Joined: Apr 14, 2011
|
#99210
Nope! There is no requirement that all 9 variables be used. This question is just about what could happen, and while we could use more variables, we don't have to. A perfectly acceptable solution to this question would be:

Zone 1: empty
Zone 2: H
Zone 3: RRR

Get the most out of your LSAT Prep Plus subscription.

Analyze and track your performance with our Testing and Analytics Package.